Die Eichtransformation des Vektorpotentials multipliziert die Wellenfunktion mit der Phase

Stellen Sie sich ein Elektron in einem elektromagnetischen Feld mit Skalar- und Vektorpotentialen vor ϕ , A . Nehmen wir der Einfachheit halber an, dass A ist zeitunabhängig. Angenommen, wir kennen die Wellenfunktion ψ dieses Elektrons. Dann ψ erfüllt

ich ψ T = [ 1 2 M ( P ^ e A C ) 2 + e ϕ ] ψ = H ^ ψ

Die Frage betrifft den Nachweis, dass bei einer Eichtransformation der Potentiale gilt:

A A ' = A + Λ
ϕ ϕ ' = ϕ 1 C Λ T = ϕ

für einige Skalare Λ ( T , X ) , transformiert sich die Wellenfunktion als

ψ ψ ' = e X P ( ich e Λ C ) ψ
dh es wird mit einer Phase multipliziert. Es ist leicht, das zu zeigen ψ ' erfüllt die transformierte Schrödinger-Gleichung:
ich ψ T ' = H ^ ' ψ '

Ich würde jedoch gerne wissen, ob es andere mögliche Lösungen für die obige Gleichung gibt. Wenn ja, welche sind das? Oder ist ψ ' die einzige Lösung?

Ich habe versucht, andere Lösungen zu finden, indem ich annahm ψ ' = F ψ , Wo F unbekannt ist, und diese dann in die neue Schrödinger-Gleichung einsetzen. Dies ergibt eine neue Differentialgleichung für F . Meine Versuche, diese Differentialgleichung zu lösen, sind jedoch bisher gescheitert.

Es gibt wahrscheinlich eine andere Möglichkeit (vielleicht über Pfadintegrale?), Dies zu zeigen, die mir nicht bekannt ist. Könnten Sie mir bitte einen Hinweis geben?

Bitte klären Sie, ob Ihre Frage diese ist: Wenn ψ Und ψ ' jeweils die Schrödinger-Gleichungen erfüllen
(01) S C H R ( ψ , A , ϕ ) = 0 (01') S C H R ( ψ ' , A ' , ϕ ' ) = 0
Und
(02) ψ ' = e ich e Λ ψ
Dann
(03a) A ' = A + Λ (03b) ϕ ' = ϕ Λ T
Nicht genau. Meine Frage ist: Wenn wir mit (01) beginnen, dann die Transformationen (03a) und (03b) durchführen, was ist die Funktion ψ ' was (01') erfüllt. Meine Frage ändert also die Reihenfolge dessen, was Sie vorschlagen. Mit anderen Worten, ich möchte das direkt aus der Differentialgleichung (01') beweisen ψ ' muss (02) sein. Hoffentlich wird dies aus meinen Änderungen klarer.
Dies ist im relativistischen Fall wirklich leicht zu sehen, da die Gleichung im Impuls linear ist. Oder auf Lagrange – im Wesentlichen dasselbe. Ich habe ein bisschen versucht, dies in Ihrem nicht-relativistischen Fall herzuleiten, aber die Mathematik geht schlecht. Sie wollen genau den angegebenen Hamilton-Operator, richtig?

Antworten (2)

Diese Antwort ist durch den Aharonov-Bohm-Effekt motiviert und beweist, was das OP verlangt, aber im Sonderfall

(01) × A = 0 = × A ' das ist B = 0

Um die Ausdrücke zu vereinfachen:

  1. Satz

    (02) = 1 , C = 1 , e = 1 , M = 1 2

  2. Verwenden Sie einen Punkt für die partielle Ableitung in Bezug auf T

    (03) ψ ˙ ( X , T ) ψ ( X , T ) T

  3. die Abhängigkeit weglassen ( X , T ) sofern nicht anders erforderlich.

Jetzt wissen wir in Übereinstimmung mit OP, dass dies der Schrödinger-Gleichung eines Teilchens in einem elektromagnetischen Feld entspricht [ A ( X , T ) , ϕ ( X , T ) ]

(04) ich ψ ˙ = [ ( ich A ) 2 + ϕ ] ψ

wir ersetzen die Wellenfunktion ψ ( X , T ) von

(05) ψ ' ( X , T ) = e ich Λ ( X , T ) ψ ( X , T ) das ist die Substitution ψ e ich Λ ψ '

dann gehorcht diese neue Wellenfunktion der Schrödinger-Gleichung eines Teilchens im elektromagnetischen Feld [ A ' ( X , T ) , ϕ ' ( X , T ) ]

(06) ich ψ ' ˙ = [ ( ich A ' ) 2 + ϕ ' ] ψ '

Wo

(07a) A ' = A + Λ , mit Λ ( X , T ) R (07b) ϕ ' = ϕ Λ ˙

Das ist zusammengefasst

(08) ( ich ψ ˙ = [ ( ich A ) 2 + ϕ ] ψ ψ ' ( X , T ) = e ich Λ ( X , T ) ψ ( X , T ) ) ( ich ψ ' ˙ = [ ( ich A ' ) 2 + ϕ ' ] ψ ' A ' = A + Λ , ϕ ' = ϕ Λ ˙ )

Hinweis: Beweise für diese Aussage finden sich in Lehrbüchern und im Internet: http://www.physicspages.com/2013/02/01/electrodynamics-in-quantum-mechanics-gauge-transformations/

Die Frage ist in ihrer 2. Version wie im Kommentar von RPF die Umkehrung von (08) im folgenden Sinne:

(09) ( ich ψ ˙ = [ ( ich A ) 2 + ϕ ] ψ ich ψ ' ˙ = [ ( ich A ' ) 2 + ϕ ' ] ψ ' A ' = A + Λ , ϕ ' = ϕ Λ ˙ ) ??? ( ψ ' ( X , T ) = e ich M ( X , T ) ψ ( X , T ) M ( X , T ) R )

Nun, wenn ψ ( X , T ) gehorcht dann (04) unter der Bedingung (01).

(10) ψ ( X , T ) = ψ 0 ( X , T ) exp [ ich Γ A ( X ' , T ) D X ' ]

Wo Γ ( X ) charakterisiert eine beliebige Kurve im dreidimensionalen Raum, die von einem beliebigen konstanten Punkt ausgeht X 0 und endet am Punkt X , wie in Abbildung, und ψ 0 ( X , T ) stellt eine Lösung der Schrödinger-Gleichung (04) dar A = 0 aber sonst willkürlich ϕ ( X , T ) , das heißt, gehorcht der reduzierten Schrödinger-Gleichung

(11) ich ψ ˙ 0 = [ ( ich ) 2 + ϕ ] ψ 0

Gleiches gilt nach der Transformation (07) und da die neue Wellenfunktion (06) dann unter der noch gültigen Bedingung (01) gehorcht

(12) ψ ' ( X , T ) = ψ 0 ' ( X , T ) exp [ ich Γ ' A ' ( X ' , T ) D X ' ]

Wo Γ ' ( X ) charakterisiert eine beliebige Kurve im dreidimensionalen Raum, die von einem beliebigen konstanten Punkt ausgeht X 0 ' und endet am Punkt X , wie in Abbildung, und ψ 0 ' ( X , T ) stellt eine Lösung der Schrödinger-Gleichung (06) dar A ' = 0 aber sonst willkürlich ϕ ' ( X , T ) [ = ϕ ( X , T ) Λ ˙ ( X , T ) ] , das heißt, gehorcht der reduzierten Schrödinger-Gleichung

(13) ich ψ ' ˙ 0 = [ ( ich ) 2 + ϕ ' ] ψ 0 '

Lassen Sie nun die Messgerät-Transformation

(14) ( ich ψ ˙ 0 = [ ( ich 0 ) 2 + ϕ ] ψ 0 ξ ( X , T ) = e ich Λ ( X , T ) ψ 0 ( X , T ) ) ( ich ξ ˙ = [ ( ich A ξ ) 2 + ϕ ξ ] ξ A ξ = 0 + Λ , ϕ ξ = ϕ Λ ˙ = ϕ ' )

das ist die Wellenfunktion ξ ( X , T ) gehorcht der Schrödinger-Gleichung

(15) ich ξ ˙ = [ ( ich Λ ) 2 + ϕ ' ] ξ

Auch für (15) ist die Bedingung (01) erfüllt

(16) × A ξ = × Λ = 0

also in Analogie zu den Paaren von ψ -Gleichungen (10)-(11) und ψ ' -Gleichungen (12)-(13)

(17) ξ ( X , T ) = ξ 0 ( X , T ) exp [ ich Γ ξ A ξ ( X ' , T ) D X ' ] = ξ 0 ( X , T ) exp [ ich Γ ξ Λ ( X ' , T ) D X ' ]

Wo Γ ξ ( X ) charakterisiert eine beliebige Kurve im dreidimensionalen Raum, die von einem beliebigen konstanten Punkt ausgeht X 0 ξ und endet am Punkt X , wie in Abbildung, und ξ 0 ( X , T ) stellt eine Lösung der Schrödinger-Gleichung (15) dar A ξ = 0 aber sonst willkürlich ϕ ' ( X , T ) , das heißt, gehorcht der reduzierten Schrödinger-Gleichung

(18) ich ξ ˙ 0 = [ ( ich ) 2 + ϕ ' ] ξ 0

Aber (18) für ξ 0 ( X , T ) ist identisch mit (13) für ψ 0 ' ( X , T ) damit wir die beiden Funktionen identifizieren können und so

(19) ξ 0 ( X , T ) ψ 0 ' ( X , T )
Das Kombinieren von (12), (19), (17) und der unteren Gleichung in linken Klammern in (14), das heißt ξ = exp [ ich Λ ] ψ 0 ' , wir haben

(20) ψ ' ( X , T ) = e ich M ( X , T ) ψ ( X , T ) (21) M ( X , T ) = Λ ( X , T ) + Γ ' A ' ( X ' , T ) D X ' Γ A ( X ' , T ) D X ' Γ ξ Λ ( X ' , T ) D X '

Wird der Startpunkt einer beliebigen Kurve gewählt, so ist das relative Phasenintegral pfadunabhängig, da die Vektorfunktion unter dem Integral null Curl hat. Der 1. und der letzte Term der rechten Seite von (21) ergeben

(22) Λ ( X , T ) Γ ξ Λ ( X ' , T ) D X ' = Λ ( X , T ) [ Λ ( X , T ) Λ ( X 0 ξ , T ) ] = Λ ( X 0 ξ , T )

Wenn wir wählen X 0 ' X 0 dann ergeben der 2. und 3. Term der rechten Seite von (21).

Γ ' A ' ( X ' , T ) D X ' Γ A ( X ' , T ) D X ' = Γ ' Λ ( X ' , T ) D X ' + Γ ' Γ A ( X ' , T ) D X ' 0 (23) = Λ ( X , T ) Λ ( X 0 , T )

Durch die Gleichungen (22) und (23) ergibt sich Gleichung (21).

(24) M ( X , T ) = Λ ( X , T ) Λ ( X 0 , T ) + Λ ( X 0 ξ , T )

Endlich, wenn wir uns entscheiden X 0 ξ X 0 Dann

(25) M ( X , T ) = Λ ( X , T )
Geben Sie hier die Bildbeschreibung ein

Referenz: BEISPIEL 1.6 Der Aharonov-Bohm-Effekt in "Quantum Mechanics - Special Chapters" von Walter Greiner, 1998, englische Ausgabe.

Ist das nicht ein Overkill?
@Frobenius: Kannst du bitte meine Frage beantworten unter: physical.stackexchange.com/questions/264916/…
OK. Dann antworte nicht. Bitte sagen Sie einfach (durch Kommentare), wo ich in meiner Frage falsch liege.
@Frobenius: Ich habe bereits Maxwells Abhandlung über "Ampere Force Law" ausgegraben. Nun möchte ich etwas wissen, was in Maxwells Abhandlung nicht im Detail steht; das ist ungefähr Newtons drittes Gesetz in schwacher Form (gleiche Wirkungsreaktion) in den vier Kraftgleichungen. Irgendwo liege ich vielleicht falsch in meiner Frage. Deshalb komme ich am Ende zu der falschen Schlussfolgerung, dass es in allen vier Fällen eine gleiche und entgegengesetzte Kraft geben würde. Ich brauche den Antwortenden, um darauf hinzuweisen, wo ich falsch liege.
@Frobenius: Ich plane, die Herleitung des Ampereschen Kraftgesetzes in einer leicht verständlichen Sprache auf eine Website zu stellen. Ich habe über zwei Monate im Internet nach einer Herleitung des Ampereschen Kraftgesetzes gesucht und konnte sie nur in Maxwells Abhandlung finden. Es könnte für diejenigen nützlich sein, die eine Ableitung des Kraftgesetzes von Ampere wissen möchten. Diese vergessenen Theorien des Elektromagnetismus müssen wiederbelebt werden.

Ich kann das rückwärts zeigen und dann die Motivation erklären. Die Eindeutigkeit der Lösung folgt aus der Bedingung, dass die Transformation unphysikalisch sein muss.

Angenommen, wir nehmen Ihre Gleichung und transformieren sie ψ :

ich T ψ = ( Π 2 2 M + e ϕ ) ψ

wird

ich T ( e ich e C Λ ψ ) = ( Π 2 2 M + e ϕ ) e ich e C Λ ψ

Lassen Sie uns dies schrittweise bewerten.

ich T ( e ich e C Λ ψ ) = ich T ( e ich e C Λ ) ψ + ich e ich e C Λ T ψ = e ich e C Λ ( e C T Λ ψ + ich Λ T ψ )

Betrachten Sie nun die Wirkung von Π ^ = P ^ e C A auf dem Exponenten.

P ^ e ich e C Λ = ich e ich e C Λ = e ich e C Λ ( Λ + P ^ )

(aufgrund der Leibnitzschen Ableitungsregel)

Indem Sie einige Zeit damit herumspielen, können Sie das zeigen

Π ^ 2 e ich e C Λ = e ich e C Λ ( Π ^ + e C Λ ) 2

Hier sieht man schon wie e ich e C Λ auf beiden Seiten der Gleichung aufhebt und wie die Additionen absorbiert werden ϕ Und A . Grundsätzlich haben wir den Exponenten mit Differentialoperatoren ausgetauscht, um dies zu erreichen.


Das Grundprinzip

Eichsymmetrie war aus der klassischen Elektrodynamik bekannt, also musste sie in die Quantenmechanik einfließen. Es ist jedoch a priori unklar, wie die elektromagnetischen Potentiale in die Gleichung eingehen sollen. Die Inspiration für die obige Gleichung stammt aus der klassischen Mechanik, wo die korrekten Bewegungsgleichungen mit der Form des Hamilton-Operators erreicht werden können, den wir sehen (z. B. hier ; die Eichinvarianz wird implizit verwendet).

Aber wenn wir die Eichtransformation direkt anwenden, bekommen wir ein Durcheinander in unserer Schrödinger-Gleichung. Aber wir gehen davon aus, dass diese Transformation keine physikalischen Konsequenzen hat – also müssen alle Observablen gleich bleiben. Der Zustand des Systems wird durch seine Wellenfunktion definiert, aber nur die Amplitude ist physikalisch, während seine Phase nicht beobachtbar ist.

Wir wollen also überschüssige Terme in der Gleichung in die koordinatenabhängige Phasenverschiebung der Wellenfunktion aufnehmen, die uns keine andere Wahl lässt als die obige Transformation.

Dies ist also die einzige Transformation, die die Amplitude nicht beeinflusst.